LSAT and Law School Admissions Forum

Get expert LSAT preparation and law school admissions advice from PowerScore Test Preparation.

 Administrator
PowerScore Staff
  • PowerScore Staff
  • Posts: 8917
  • Joined: Feb 02, 2011
|
#39063
Complete Question Explanation
(The complete setup for this game can be found here: lsat/viewtopic.php?t=15163)

The correct answer choice is (A)

This question asks you to determine what could be true if G gives the S lecture. As with most Could Be True questions, the process of elimination is likely to be key: any answer choice that cannot be true will be incorrect.

If G gives the S lecture, we need to work with Template 1B only. There are two solutions possible, depending on whether the GS block is third or fourth. Thus, two local setups are in order:
PT72 - Game_#2_#10_diagram 1.png
Answer choice (A) is the correct answer choice, because L could be delivered third according to one of the two solutions outlined above.

Answer choice (B) is incorrect, because O must be second, not third.

Answer choice (C) is incorrect, because the first lecture must be W, not S.

Answer choice (D) is incorrect, because the second lecture must be O, not S.

Answer choice (E) is incorrect, because W must be first, not second.
You do not have the required permissions to view the files attached to this post.
 rachelbernard
  • Posts: 6
  • Joined: Jul 07, 2020
|
#77196
When I attempted this question on a PT, I didn't choose to do templates (and your post on the setup agrees that templates weren't necessary to this game). Can someone show how you would attack this question without the use of premade templates? I ran out of time later in the section, and am wondering if I could have answered questions like this more efficiently.
 Luke Haqq
PowerScore Staff
  • PowerScore Staff
  • Posts: 742
  • Joined: Apr 26, 2012
|
#77208
Hi rachelbernard,

Great question!

While identifying the templates can be a great time-saving strategy for certain games, many times that might not be what people actually do when taking an actual LSAT. I can explain in this post how one could attack the question without templates, specifically by explaining with reference to the final diagram after inferences have been made from the rules. I would also encourage you to visit the link provided in the prior post--that thread goes through a step-by-step explanation of how to incorporate each rule into the game diagram and how to make all the possible inferences from the rules. That provides one with the following final diagram:

Hist.: | F/H | H/F | G/J | J/G |

Lect.: | S/W | S/W/O | ___ | L/S |

Again, for a better picture of this diagram, check out that link with the walk-through of the setup for the game. We can use this diagram, however, to address question 10.

That question asks what could be true if Garcia gives the sculptures lecture. It therefore gives us a vertical G-S block. Based on the inferences from the original rules, we know that this block can only occur in the 3rd or 4th time slots. Looking to the answer choices, a useful strategy to apply first is to see whether any of them cannot be true off the bat, based on the final diagram from the original rules alone. Unfortunately, in this case it doesn't allow one to eliminate any of the answer choices, but it could still be useful for that reason--i.e., it indicates how there is a lot of room for how the lecture topics are ordered, while there might be more constraints when these are tied to a particular speaker.

Since that strategy wouldn't eliminate answer choices, a next step would be to plug in the G-S block. Even if one did not diagram this as an identify-the-templates game at the stage of working with the original rules, a next strategy for eliminating answer choices would be to input the block that question 10 provides. As mentioned, it can only occur in one of two places, so it wouldn't be too time-consuming to try both. Suppose the G-S block occurred in the 3rd time slot:

Hist.: | F/H | H/F | G | J/G |

Lect.: | S/W | S/W/O | S | L/S |

If that were the case, we could eliminate G and S as contenders for other spots, allowing us to fill in several gaps:

Hist.: | F/H | H/F | G | J |

Lect.: | W | O | S | L |

Based on this placement of the G-S block, nearly all of the variables are locked into place, and the only uncertainty concerns F and H. One would thus want to look to those two variables in the answer choices. Again, however, this wouldn't help eliminate any, since all the answer choices concern the lectures rather than the historians.

This would bring one to a second diagram, this time with the G-S block in the 4th time slot rather than the third:

Hist.: | F/H | H/F | G/J | G |

Lect.: | S/W | S/W/O | ___ | S |

Having that block does help us place some of the other variables:

Hist.: | F/H | H/F | J | G |

Lect.: | W | O | L | S |

As it turns out, the only uncertainty we're left with again appears to be only the placement of F and H. However, note that comparing these two placements of the G-S block together helps eliminate 4 incorrect answer choices and points to the correct answer. If one looks at both of them, none of answer choices (B) through (E) is possible, so they are incorrect for a could be true question. By contrast, in one of these setups (the second one, with the G-S block in the 4th time slot), the lithographs lecture is delivered 3rd, making (A) the correct answer choice.

Get the most out of your LSAT Prep Plus subscription.

Analyze and track your performance with our Testing and Analytics Package.